You are on page 1of 5

1

Outline of solutions to Homework 5


Problem 7.1: a) We set the voltage of the inner spherical shell to be V and the potential of the outer one to be
0, so the voltage difference is V as required. By symmetry, the current density is directed radially outward. If I is
the total current, then the current density is
~j(r) =

I
r.
4r2

(1)

Inside the conductor the voltage obeys Laplaces equation. The system is spherically symmetric so the solution to
Laplaces equation in spherical polars reduces to just the l = 0 term, ie, V (r) = A0 + B0 /r. We then have (assuming
no polarization effects),
A0 + B0 /a = V ;

A0 + B0 /b = 0

so that

B0 =

abV
;
ba

A0 =

aV
ba

and V =

aV b
[ 1]
ba r

(2)

The electric field is then,


V
aV b
~
r
E(r)
=
=
[ ]
r
b a r2

(3)

~ we find that the magnitude of the ~j is,


Using ~j = E,
j(r) = E(r) =
b) Using V = IR, we then find R =

ba
4ab .

aV b
I
[ ]=
b a r2
4r2

so that

I = 4

abV
ba

(4)

This can also be derived by using the series combination of shells,


b

Z
R=
a

c) In the limit of large b, the resistance is R =

2
4a ,

ba
dr
=
4r2
4ab

(5)

so the current is I = V /R = 2aV .

Problem 7.2: a) Summing the voltages around the loop, we have,


VC + VR = 0 =

Q
Q
dQ
+ iR =
+R
C
C
dt

so that

Q(t) = Q0 et/RC

(6)

where Q0 = CV0 . The current is


i=

dQ
= (Q0 /RC)et/RC = (V0 /R)et/RC
dt

b) The energy stored in the capacitor is CV02 /2, while the energy found from the dissipated power is,
Z
Z
RC V0 2
1
V0
)( ) |0 = CV02
U=
i2 Rdt =
( )2 e2t/RC dt = (
R
2
R
2
0
0

(7)

(8)

c) The circuit equation is now,


V0 + VC + VR = 0 = V0

Q
dQ
Q
iR = V0 R
C
C
dt

so that

Q(t) = Q0 (1 et/RC )

The current is dQ/dt = (V0 /R)et/RC . The total energy supplied by the battery is,
Z
Z
V02
V0 Idt = ( )
et/RC dt = CV02
R 0
0

(9)

(10)

The energy dissipated in the resistor is the same as that calculated for the discharge case. Therefore one half of the
power is dissipated in the resistor, no matter how small or large the resistance is.
Problem 7.7: We have a metal bar on conducting rails moving through a region of constant magnetic field into
the page. The loop has resistance R. a) The motional emf is Blv, so the current is Blv/R upward in the bar in Fig.

2
7.16. b) The magnetic drag force is ilB, opposite to the direction of motion. ilB = B 2 l2 v/R c) Newtons second law
gives,
Fdrag =
where =

B 2 l2
mR .

B 2 l2
dv
v=m
R
dt

so that

v(t) = v0 et

(11)

c) The energy delivered to the resistor is,


Z
U=

i2 (t)Rdt =

(Bl)2
R

v 2 dt =

(Blv0 )2
R

e2t dt =

1
mv 2
2 0

(12)

Problem 7.8: a) The magnetic field due to the wire is B(r) = 0 i/(2s) out of the page. The flux through the
square loop is,
Z
Z s+a
0 i
0 ia
B = Bdsdz = a
=
ln(1 + a/s)
(13)
2s
2
s
b) When the loop is pulled away from wire, the flux (out of page) decreases. Current is induced to increase the flux
(out of page) in the loop. The induced current in the loop is then counterclockwise. The emf generated is
E =

dB
d 0 ia
0 ia s a ds
= [
ln(1 + a/s)] =
dt
dt 2
2 a + s s2 dt

(14)

Using v = ds/dt, this reduces to


E=

0 i a2 v
2 s(a + s)

(15)

c) If the loop is pulled to the right, the flux through the loop remains constant so there is no induced emf.
Problem 7.11: The motional emf is Blv and the magnetic drag force is (Bl)2 v/R upward. Newtons second law
is then
mg Fdrag = m

dv
dt

or

dv (Bl)2
+
vg =0
dt
mR

(16)

The resistance of the loop is R = 4lr /a, where r is the resistivity of aluminum, l is the total perimeter of the loop
and a is the crossectional area of the wires making up the loop. The mass of the loop is 4alm , where m is the
density of aluminum (2700kg/m3 ). Therefore,
(Bl)2
(Bl)2 a
B2
=
=
=b
mR
a16l2 r m
16m r

(17)

Solving the equation we have


v(t) = v (1 ebt )

where

v = g/b =

16gm r
B2

(18)

b is the inverse of the relaxation time. For B = 1T , g = 9.8m/s2 , m = 2700kg/m3 and conductivity
= 1/r = 1/(2.65 108 )(m)1 . Plugging the numbers gives v = 1.12cm/s. The relaxation time is
1/b 1.15ms. This is the time require to reach 1 1/e of the terminal velocity (about 63%). If the loops were cut,
there could be no current and hence no magnetic drag force. The loop would fall under gravity in the usual way.
~ = ky 3 t2 z. The magnetic flux
Problem 7.13: Square loop of side a in the first quadrant of xy plane, with field B
through the loop is,
Z a
1
B = akt2
y 3 dy = ka5 t2
(19)
4
0
The magnitude of the induced emf is dB /dt = ka5 t/2. For positive k, The flux is increasing in the positive z
direction, so the induced flux opposes this increase. Therefore for positive k the induced emf in the loop is clockwise

3
in the x-y plane.
Problem 7.17: a) The magnetic field in the solenoid is B = nI0 , so the flux in the solenoid is B = ni0 a2 .
The magnitude of the induced emf in the loop is then nk0 a2 . The flux through the solenoid is to the right and
increasing (for k positive), so the induced current in the loop is opposite that of the solenoid. The induced current
then flows to the right through the resistor and has magnitude nk0 a2 /R. b) The total charge that flows is
Z
Z
1 dB
2B
2nI0 a2
idt =
Q=
dt =
=
(20)
R
R
R dt

Problem 7.20: Definition of mutual inductance: 2 = M21 i1 (when i2 = 0), 1 = M12 i2 (when i1 = 0), Selfinductance is defined through B = Li. a) Assume that the small loop is a long way from the large loop so the
magnetic field due to the large loop is essentially constant and is that on the z axis of a circular ring (see Lecture
18, Eq. 9), then,
s2
~ big = 0 ibig
B
2 (s2 + z 2 )3/2

so that

small = M21 ibig ,

with M21 =

0 Areasmall
b2
2
(b2 + z 2 )3/2

(21)

where Areasmall = a2 . Now lets find the flux coupling using Bsmall as the source. In that case we can approximate
the field as that of a dipole, so that,

~ small = 0 m [2cos
B
r + sin]
4r3

(22)

We can choose a convenient surface to calculate the flux through the big loop - the spherical cap bounded by the big
loop, so that,
Z
Z 2 Z cap
0 im
0 im
~
big = Bsmall da
r=
(2cos)R2 sin()dd =
sin2 (cap )
(23)
3
4R
4R
0
0
where R2 = z 2 + b2 and sin(cap ) = b/R. Therefore, This reduces to,
0 ismall Areasmall b2
= M12 ismall
2(z 2 + b2 )3/2

big =

(24)

Comparing Eq. (21) and Eq. (24) we see that M21 = M12 .
Problem 7.23: The self-inductance is found from B = Li. The magnitude of the magnetic field within the loop
1
0i 1
in Fig. 7.37 is B(s) = 2
[ s + ds
], so the magnetic flux within the loop is,
d

Z
B = l

2


0 il
0 i 1
ds =
ln[(d )/]
2 s

(25)

This diverges as  0 due to the divergence of the magnetic field. In reality the field does not diverge in a real wire
and this divergence is due to the mathematical simplification of a very narrow wire.
Problem 7.27: The field of a toroid with center radius R, and circular cross-section of radius b is found by
assuming the the field only depends on r, so that Boutside = 0. Then the field inside is found from,
2rB(|r R| < b) = 0 N i

so that

Binside =

0 N i
2r

(26)

The energy stored in the toroid is,


W =

1
h
20

2r(
a

0 N i 2
0 N 2 i2
) dr =
hln(b/a)
2r
4

(27)

This work should be equal to the energy found using the formula Li2 /2 and with the formula Eq. (7.27), we have,
U=

0 N 2 h
ln(b/a)i2
4

(28)

4
demonstrating the equivalence of the two methods.
Problem 7.54 a) The definitions of L1 , L2 , M are,
N1 1 = L1 i1 + M i2 ;

N2 2 = L2 i2 + M i1 ;

(29)

N1 1 = L1 i1 ;

N2 2 = M i 1 ;

(30)

N2 2 = L2 i2 ;

N1 1 = M i 2 ;

(31)

When i2 = 0 we have,

while when i1 = 0,

Since L1 , L2 , M are constants, we can find them from any of these equations. Eliminating the flux from the latter
two sets yields M i2 = L1 i1 ; M i1 = L2 i2 and hence M 2 = L1 L2 .
b) Taking a time derivative of Eqs. (29) above and using both Faradays law and Kirchhoffs voltage law, we have,
V1 cos(t) = L1

dI1
dI2
+M
;
dt
dt

I2 R = L2

dI2
dI1
+M
dt
dt

(32)

c) Using the second of these equations to eliminate I1 from the first equation also leads to cancellation of the time
derivative term in I2 in the first equation. We can then solve for
I2 (t) =

V1 L2
cos(t)
MR

(33)

Substituting this back into the second of equations (32) and solving yields,
I1 (t) =

V1 1
L2
[ sin(t) +
cos(t)]
L1
R

(34)

d) Taking the ratio I2 R/Vin gives,


V1ML2 cos(t)
L2
=
V1 cos(t)
M

(35)

Noting that for an ideal transformer 1 = 2 Eq. (31) gives L2 /M = N2 /N1 and hence the result quoted. The minus
sign indicates that the input and out are out of phase.
e) Show that the time average of Vin I1 = Vout I2 = I22 R.
Vin I1 =

V1 1
L2
[ sin(t) +
cos(t)]V1 cos(t)
L1
R

(36)

The time average corresponds to an average of a period of this function. The average of sin(x)cos(x) over a period
is zero, while the time average of cos2 (x) over a period is T /2, where T is the period. In our case = 2/T , so the
average over a period gives, and hence the time average of,
< Vin I1 >t =

V12 L2
L1 R

(37)

We also have,
< Vout I2 >t =< I22 R >t =< (

V1 L2
V12 L22
V12 L2
cos(t))2 R >t =
=
MR
M 2R
L1 R

(38)

Problem 9.9 a) A wave polarized in the z direction and travelling in the negative x direction has electric
~ = E0 zcos(kx + t) and its propagation vector is ~k = k
field, E
x. Its magnetic field is then
~ = 1 k E
~ = E0 ycos(kx + t)
B
c
c

(39)

5
b) In this case ~k =

+ y + z), with = c|~k|.. The polarization is in the x z plane, so we have n


= a
x + b
z.
1

However we know that n


k = 0, so that a = b. Normalization of n
then yields, n
= 2 (
x z). Note that both n

and
n are correct. The electric field is given by,
k

(
x
3

1
k
~ = E0 n
E
cos(~k ~r t) = E0 (
x z)cos( (x + y + z) t)
2
3

(40)

1
k
~ = 1 k E
~ = E0 1 (
B
x + y + z) (
x z)cos( (x + y + z) t)
c
c 3
2
3

(41)

k
E0
~ =
(
x + 2
y z)cos( (x + y + z) t)
B
6c
3

(42)

The magnetic field is,

which reduces to

You might also like